LSAT and Law School Admissions Forum

Get expert LSAT preparation and law school admissions advice from PowerScore Test Preparation.

User avatar
 Dave Killoran
PowerScore Staff
  • PowerScore Staff
  • Posts: 5852
  • Joined: Mar 25, 2011
|
#88142
Complete Question Explanation
(The complete setup for this game can be found here: lsat/viewtopic.php?f=167&p=88139#p88139)

The correct answer choice is (C).

The question stem specifies that G is the second passenger to exit, and that L and S are the first two stops, not necessarily in that order:

G4-Q22-d1.png

Of course, if L and S are the first two stops, then F and M must be the last two stops, in some order. If M is third or fourth, then from the second rule we can infer that R cannot be first, and must be third or fourth. From the third rule J cannot be first, and so the only passenger who is available to exit first is V. With V exiting first, J and R must exit third and fourth, not necessarily in that order. Thus, without accounting entirely for the second and fourth rules, this is the current setup for this question:

G4-Q22-d2.png

While the actions of the second and fourth rules limit the number of solutions possible under the scenario above, the information thus far is sufficient to answer this question. Because S is the first or second stop, and R must exit third or fourth, answer choice (C) must be true and is correct.

The following hypothetical eliminates answer choices (A) and (D):

G4-Q22-d3.png

The following hypothetical eliminates answer choice (B):

G4-Q22-d4.png

The following hypothetical eliminates answer choice (E):

G4-Q22-d5.png
You do not have the required permissions to view the files attached to this post.
 josuecarolina
  • Posts: 24
  • Joined: Jul 20, 2012
|
#22138
Hi,

Can you explain #22?

Thank you!
 Nicholas Bruno
PowerScore Staff
  • PowerScore Staff
  • Posts: 62
  • Joined: Sep 27, 2011
|
#22139
Hi,
First, I would recommend that you read a prior post about the setup lsat/viewtopic.php?f=167&t=8582.


22) We know that G is not greater than S so J cannot be greater than F. We also know that G is second.

With G second, we know that L and S are the first two stops since G has to be on the bus at both stops. This leads us to know that R must be third or forth since it must not be before M. We know that J cannot be first (since V must come after it). This leaves only V for the first spot.

The diagram would look like:

V G R/J R/J
L/S L/S F/M F/M

A is wrong: V is not necessarily on the bus when the bus gets to S.
B is wrong: same as A
C: correct - we know R will be on the bus until S has passed
D: we do not know this - R could get off at M - with both at stop 3
E: same explanation as D.

Let me know if this helps you out!
 eober
  • Posts: 107
  • Joined: Jul 24, 2014
|
#22130
Hi,

How did we set up the template in this game. I had a very poorly constructed template for this game and I couldn't solve #22. Could you explain what the best way to diagram this game would be?

Thanks!
 David Boyle
PowerScore Staff
  • PowerScore Staff
  • Posts: 836
  • Joined: Jun 07, 2013
|
#22131
Hello,

One way to set it up:

la :arrow: 1,2 (or putting la/, /la over spaces 1, 2)
m :arrow: r still on board (though she could presumably get off at m)
v > j (v gets off before j)
f + j on board :dbl: s + g on board

For question 22, since la and s must be in the first two, if g gets off second, then m must be in space 3 or 4. So r must still be on board at s, since she has to get off later, at m or after. (Answer C)


Hope that helps,
David
 eober
  • Posts: 107
  • Joined: Jul 24, 2014
|
#22132
Makes perfect sense, thanks!
 angelsfan0055
  • Posts: 39
  • Joined: Feb 26, 2021
|
#91591
Hi, this question was difficult for me, and I'm not sure I'm grasping the explanation. how do we know from the Stem that Greg is second? I also was not sure if this was a local or global question based on how it was worded. I ended up choosing B based on a previous hypothetical
 Adam Tyson
PowerScore Staff
  • PowerScore Staff
  • Posts: 5153
  • Joined: Apr 14, 2011
|
#91603
The stem ends with this phrase, angelsfan0055: "assuming he (Greg) is the second one off the van?" That's how we know Greg is second in this question - the stem said to assume he was!

And that assumption, along with the earlier "if" phrase ("Which one of the following must be true if.. "), is what tells us that this is a local question. When the question creates new rules and restrictions that were not in the original rules, that's what makes it what we call "local." The restrictions set forth in the question stem apply only to this question, and they do not change the original rules.

Get the most out of your LSAT Prep Plus subscription.

Analyze and track your performance with our Testing and Analytics Package.